1.17 problem 17

1.17.1 Solved as first order quadrature ode
1.17.2 Solved as first order Exact ode
1.17.3 Maple step by step solution
1.17.4 Maple trace
1.17.5 Maple dsolve solution
1.17.6 Mathematica DSolve solution

Internal problem ID [3718]
Book : Advanced Mathematica, Book2, Perkin and Perkin, 1992
Section : Chapter 11.3, page 316
Problem number : 17
Date solved : Thursday, October 17, 2024 at 04:22:21 AM
CAS classification : [_quadrature]

Solve

\begin{align*} \left (x^{3}+1\right ) y^{\prime }&=3 x^{2} \tan \left (x \right ) \end{align*}

With initial conditions

\begin{align*} y \left (0\right )&=\frac {\pi }{2} \end{align*}

1.17.1 Solved as first order quadrature ode

Time used: 0.239 (sec)

Since the ode has the form \(y^{\prime }=f(x)\), then we only need to integrate \(f(x)\).

\begin{align*} \int {dy} &= \int {\frac {3 x^{2} \tan \left (x \right )}{x^{3}+1}\, dx}\\ y &= \int \frac {3 x^{2} \tan \left (x \right )}{x^{3}+1}d x + c_1 \end{align*}
\begin{align*} y&= \int _{x_0}^{x} \frac {3 \tau ^{2} \tan \left (\tau \right )}{\tau ^{3}+1}\,d \tau \, + y_0 \\ y&= \int _{0}^{x}\frac {3 \tau ^{2} \tan \left (\tau \right )}{\tau ^{3}+1}d \tau +\frac {\pi }{2} \end{align*}
Figure 60: Slope field plot
\(\left (x^{3}+1\right ) y^{\prime } = 3 x^{2} \tan \left (x \right )\)
1.17.2 Solved as first order Exact ode

Time used: 0.135 (sec)

To solve an ode of the form

\begin{equation} M\left ( x,y\right ) +N\left ( x,y\right ) \frac {dy}{dx}=0\tag {A}\end{equation}

We assume there exists a function \(\phi \left ( x,y\right ) =c\) where \(c\) is constant, that satisfies the ode. Taking derivative of \(\phi \) w.r.t. \(x\) gives

\[ \frac {d}{dx}\phi \left ( x,y\right ) =0 \]

Hence

\begin{equation} \frac {\partial \phi }{\partial x}+\frac {\partial \phi }{\partial y}\frac {dy}{dx}=0\tag {B}\end{equation}

Comparing (A,B) shows that

\begin{align*} \frac {\partial \phi }{\partial x} & =M\\ \frac {\partial \phi }{\partial y} & =N \end{align*}

But since \(\frac {\partial ^{2}\phi }{\partial x\partial y}=\frac {\partial ^{2}\phi }{\partial y\partial x}\) then for the above to be valid, we require that

\[ \frac {\partial M}{\partial y}=\frac {\partial N}{\partial x}\]

If the above condition is satisfied, then the original ode is called exact. We still need to determine \(\phi \left ( x,y\right ) \) but at least we know now that we can do that since the condition \(\frac {\partial ^{2}\phi }{\partial x\partial y}=\frac {\partial ^{2}\phi }{\partial y\partial x}\) is satisfied. If this condition is not satisfied then this method will not work and we have to now look for an integrating factor to force this condition, which might or might not exist. The first step is to write the ODE in standard form to check for exactness, which is

\[ M(x,y) \mathop {\mathrm {d}x}+ N(x,y) \mathop {\mathrm {d}y}=0 \tag {1A} \]

Therefore

\begin{align*} \left (x^{3}+1\right )\mathop {\mathrm {d}y} &= \left (3 x^{2} \tan \left (x \right )\right )\mathop {\mathrm {d}x}\\ \left (-3 x^{2} \tan \left (x \right )\right )\mathop {\mathrm {d}x} + \left (x^{3}+1\right )\mathop {\mathrm {d}y} &= 0 \tag {2A} \end{align*}

Comparing (1A) and (2A) shows that

\begin{align*} M(x,y) &= -3 x^{2} \tan \left (x \right )\\ N(x,y) &= x^{3}+1 \end{align*}

The next step is to determine if the ODE is is exact or not. The ODE is exact when the following condition is satisfied

\[ \frac {\partial M}{\partial y} = \frac {\partial N}{\partial x} \]

Using result found above gives

\begin{align*} \frac {\partial M}{\partial y} &= \frac {\partial }{\partial y} \left (-3 x^{2} \tan \left (x \right )\right )\\ &= 0 \end{align*}

And

\begin{align*} \frac {\partial N}{\partial x} &= \frac {\partial }{\partial x} \left (x^{3}+1\right )\\ &= 3 x^{2} \end{align*}

Since \(\frac {\partial M}{\partial y} \neq \frac {\partial N}{\partial x}\), then the ODE is not exact. Since the ODE is not exact, we will try to find an integrating factor to make it exact. Let

\begin{align*} A &= \frac {1}{N} \left (\frac {\partial M}{\partial y} - \frac {\partial N}{\partial x} \right ) \\ &=\frac {1}{x^{3}+1}\left ( \left ( 0\right ) - \left (3 x^{2} \right ) \right ) \\ &=-\frac {3 x^{2}}{x^{3}+1} \end{align*}

Since \(A\) does not depend on \(y\), then it can be used to find an integrating factor. The integrating factor \(\mu \) is

\begin{align*} \mu &= e^{ \int A \mathop {\mathrm {d}x} } \\ &= e^{\int -\frac {3 x^{2}}{x^{3}+1}\mathop {\mathrm {d}x} } \end{align*}

The result of integrating gives

\begin{align*} \mu &= e^{-\ln \left (x^{3}+1\right ) } \\ &= \frac {1}{x^{3}+1} \end{align*}

\(M\) and \(N\) are multiplied by this integrating factor, giving new \(M\) and new \(N\) which are called \(\overline {M}\) and \(\overline {N}\) for now so not to confuse them with the original \(M\) and \(N\).

\begin{align*} \overline {M} &=\mu M \\ &= \frac {1}{x^{3}+1}\left (-3 x^{2} \tan \left (x \right )\right ) \\ &= -\frac {3 x^{2} \tan \left (x \right )}{x^{3}+1} \end{align*}

And

\begin{align*} \overline {N} &=\mu N \\ &= \frac {1}{x^{3}+1}\left (x^{3}+1\right ) \\ &= 1 \end{align*}

Now a modified ODE is ontained from the original ODE, which is exact and can be solved. The modified ODE is

\begin{align*} \overline {M} + \overline {N} \frac { \mathop {\mathrm {d}y}}{\mathop {\mathrm {d}x}} &= 0 \\ \left (-\frac {3 x^{2} \tan \left (x \right )}{x^{3}+1}\right ) + \left (1\right ) \frac { \mathop {\mathrm {d}y}}{\mathop {\mathrm {d}x}} &= 0 \end{align*}

The following equations are now set up to solve for the function \(\phi \left (x,y\right )\)

\begin{align*} \frac {\partial \phi }{\partial x } &= \overline {M}\tag {1} \\ \frac {\partial \phi }{\partial y } &= \overline {N}\tag {2} \end{align*}

Integrating (1) w.r.t. \(x\) gives

\begin{align*} \int \frac {\partial \phi }{\partial x} \mathop {\mathrm {d}x} &= \int \overline {M}\mathop {\mathrm {d}x} \\ \int \frac {\partial \phi }{\partial x} \mathop {\mathrm {d}x} &= \int -\frac {3 x^{2} \tan \left (x \right )}{x^{3}+1}\mathop {\mathrm {d}x} \\ \tag{3} \phi &= \int -\frac {3 x^{2} \tan \left (x \right )}{x^{3}+1}d x+ f(y) \\ \end{align*}

Where \(f(y)\) is used for the constant of integration since \(\phi \) is a function of both \(x\) and \(y\). Taking derivative of equation (3) w.r.t \(y\) gives

\begin{equation} \tag{4} \frac {\partial \phi }{\partial y} = 0+f'(y) \end{equation}

But equation (2) says that \(\frac {\partial \phi }{\partial y} = 1\). Therefore equation (4) becomes

\begin{equation} \tag{5} 1 = 0+f'(y) \end{equation}

Solving equation (5) for \( f'(y)\) gives

\[ f'(y) = 1 \]

Integrating the above w.r.t \(y\) gives

\begin{align*} \int f'(y) \mathop {\mathrm {d}y} &= \int \left ( 1\right ) \mathop {\mathrm {d}y} \\ f(y) &= y+ c_1 \\ \end{align*}

Where \(c_1\) is constant of integration. Substituting result found above for \(f(y)\) into equation (3) gives \(\phi \)

\[ \phi = \int -\frac {3 x^{2} \tan \left (x \right )}{x^{3}+1}d x +y+ c_1 \]

But since \(\phi \) itself is a constant function, then let \(\phi =c_2\) where \(c_2\) is new constant and combining \(c_1\) and \(c_2\) constants into the constant \(c_1\) gives the solution as

\[ c_1 = \int -\frac {3 x^{2} \tan \left (x \right )}{x^{3}+1}d x +y \]

Solving for the constant of integration from initial conditions, the solution becomes

\begin{align*} \int -\frac {3 x^{2} \tan \left (x \right )}{x^{3}+1}d x +y = -3 \left (\int _{}^{0}\frac {\textit {\_a}^{2} \tan \left (\textit {\_a} \right )}{\textit {\_a}^{3}+1}d \textit {\_a} \right )+\frac {\pi }{2} \end{align*}
Figure 61: Slope field plot
\(\left (x^{3}+1\right ) y^{\prime } = 3 x^{2} \tan \left (x \right )\)
1.17.3 Maple step by step solution
\[ \begin {array}{lll} & {} & \textrm {Let's solve}\hspace {3pt} \\ {} & {} & \left [\left (x^{3}+1\right ) \left (\frac {d}{d x}y \left (x \right )\right )=3 \tan \left (x \right ) x^{2}, y \left (0\right )=\frac {\pi }{2}\right ] \\ \bullet & {} & \textrm {Highest derivative means the order of the ODE is}\hspace {3pt} 1 \\ {} & {} & \frac {d}{d x}y \left (x \right ) \\ \bullet & {} & \textrm {Solve for the highest derivative}\hspace {3pt} \\ {} & {} & \frac {d}{d x}y \left (x \right )=\frac {3 \tan \left (x \right ) x^{2}}{x^{3}+1} \\ \bullet & {} & \textrm {Integrate both sides with respect to}\hspace {3pt} x \\ {} & {} & \int \left (\frac {d}{d x}y \left (x \right )\right )d x =\int \frac {3 \tan \left (x \right ) x^{2}}{x^{3}+1}d x +\mathit {C1} \\ \bullet & {} & \textrm {Evaluate integral}\hspace {3pt} \\ {} & {} & y \left (x \right )=-\mathrm {I} \ln \left (x^{3}+1\right )-3 \,\mathrm {I} \left (\int -\frac {2 x^{2}}{\left (\left ({\mathrm e}^{\mathrm {I} x}\right )^{2}+1\right ) \left (x^{3}+1\right )}d x \right )+\mathit {C1} \\ \bullet & {} & \textrm {Use initial condition}\hspace {3pt} y \left (0\right )=\frac {\pi }{2} \\ {} & {} & \frac {\pi }{2}=-3 \,\mathrm {I} \left (\int ^{0}-\frac {2 \textit {\_a}^{2}}{\left (\left ({\mathrm e}^{\mathrm {I} \textit {\_a}}\right )^{2}+1\right ) \left (\textit {\_a}^{3}+1\right )}d \textit {\_a} \right )+\mathit {C1} \\ \bullet & {} & \textrm {Solve for}\hspace {3pt} \textit {\_C1} \\ {} & {} & \mathit {C1} =3 \,\mathrm {I} \left (\int ^{0}-\frac {2 \textit {\_a}^{2}}{\left (\left ({\mathrm e}^{\mathrm {I} \textit {\_a}}\right )^{2}+1\right ) \left (\textit {\_a}^{3}+1\right )}d \textit {\_a} \right )+\frac {\pi }{2} \\ \bullet & {} & \textrm {Substitute}\hspace {3pt} \textit {\_C1} =3 \,\mathrm {I} \left (\int ^{0}-\frac {2 \textit {\_a}^{2}}{\left (\left ({\mathrm e}^{\mathrm {I} \textit {\_a}}\right )^{2}+1\right ) \left (\textit {\_a}^{3}+1\right )}d \textit {\_a} \right )+\frac {\pi }{2}\hspace {3pt}\textrm {into general solution and simplify}\hspace {3pt} \\ {} & {} & y \left (x \right )=-\mathrm {I} \ln \left (x^{3}+1\right )+6 \,\mathrm {I} \left (\int \frac {x^{2}}{\left ({\mathrm e}^{2 \,\mathrm {I} x}+1\right ) \left (x^{3}+1\right )}d x \right )-6 \,\mathrm {I} \left (\int ^{0}\frac {\textit {\_a}^{2}}{\left ({\mathrm e}^{2 \,\mathrm {I} \textit {\_a}}+1\right ) \left (\textit {\_a}^{3}+1\right )}d \textit {\_a} \right )+\frac {\pi }{2} \\ \bullet & {} & \textrm {Solution to the IVP}\hspace {3pt} \\ {} & {} & y \left (x \right )=-\mathrm {I} \ln \left (x^{3}+1\right )+6 \,\mathrm {I} \left (\int \frac {x^{2}}{\left ({\mathrm e}^{2 \,\mathrm {I} x}+1\right ) \left (x^{3}+1\right )}d x \right )-6 \,\mathrm {I} \left (\int ^{0}\frac {\textit {\_a}^{2}}{\left ({\mathrm e}^{2 \,\mathrm {I} \textit {\_a}}+1\right ) \left (\textit {\_a}^{3}+1\right )}d \textit {\_a} \right )+\frac {\pi }{2} \end {array} \]

1.17.4 Maple trace
`Methods for first order ODEs: 
--- Trying classification methods --- 
trying a quadrature 
<- quadrature successful`
 
1.17.5 Maple dsolve solution

Solving time : 0.050 (sec)
Leaf size : 37

dsolve([(x^3+1)*diff(y(x),x) = 3*tan(x)*x^2, 
       op([y(0) = 1/2*Pi])],y(x),singsol=all)
 
\[ y \left (x \right ) = 3 \left (\int _{0}^{x}\frac {\tan \left (\textit {\_z1} \right ) \textit {\_z1}^{2}}{\left (\textit {\_z1} +1\right ) \left (\textit {\_z1}^{2}-\textit {\_z1} +1\right )}d \textit {\_z1} \right )+\frac {\pi }{2} \]
1.17.6 Mathematica DSolve solution

Solving time : 7.841 (sec)
Leaf size : 35

DSolve[{(1+x^3)*D[y[x],x]==3*x^2*Tan[x],y[0]==Pi/2}, 
       y[x],x,IncludeSingularSolutions->True]
 
\[ y(x)\to \int _0^x\frac {3 K[1]^2 \tan (K[1])}{K[1]^3+1}dK[1]+\frac {\pi }{2} \]